Đến nội dung

Hình ảnh

Đề thi TS trường THPT Lê Hồng Phong Nam Định 2015-2016 (2 vòng)


  • Please log in to reply
Chủ đề này có 35 trả lời

#1
HatNangNgoaiThem

HatNangNgoaiThem

    Binh nhất

  • Thành viên
  • 32 Bài viết

Đây là đề thi Toán chung của trường THPT Lê Hồng Phong Nam Định các bạn xem thử

Hình gửi kèm

  • IMG_20150602_174558.jpg

Bài viết đã được chỉnh sửa nội dung bởi HatNangNgoaiThem: 02-06-2015 - 21:23

          :icon12:  :icon12:  :icon12:  :icon12:  :icon12:  :icon12:  :icon12:  :icon12:

                    Hãy......................!!!

                          Sống chậm lại.............!!! :icon12:

                                   Nghĩ khác đi..............!!! :icon12:

                          Và yêu thương nhiều hơn.!!!!!!! @};-

                                 học cách yêu thương .....!!! :wub:  

                                                 :icon12:  :icon12:  :icon12:  :icon12:  :icon12:  :icon12:  :icon12: :icon12: :icon12:  :icon12:  


#2
Congnghiaky298

Congnghiaky298

    Binh nhất

  • Thành viên
  • 25 Bài viết

Câu 5: 2) $T=\sum \frac{1}{b^{5}c+c^5b+1}\leq \frac{1}{3b^2c^2}+\frac{1}{3a^{2}b^{2}}+\frac{1}{3a^2c^2}$

$T\leq \frac{1}{3}(a^{2}+b^2+c^2)\leq 1$

Dấu "=" xảy ra khi a=b=c=1



#3
Hoang Nhat Tuan

Hoang Nhat Tuan

    Hỏa Long

  • Thành viên
  • 974 Bài viết

Câu 5: 2) $T=\sum \frac{1}{b^{5}c+c^5b+1}\leq \frac{1}{3b^2c^2}+\frac{1}{3a^{2}b^{2}}+\frac{1}{3a^2c^2}$

$T\leq \frac{1}{3}(a^{2}+b^2+c^2)\leq 1$

Dấu "=" xảy ra khi a=b=c=1

Bạn giải thế này nó ra ngược dấu rồi  :closedeyes:


Ngài có thể trói cơ thể tôi, buộc tay tôi, điều khiển hành động của tôi: ngài mạnh nhất, và xã hội cho ngài thêm quyền lực; nhưng với ý chí của tôi, thưa ngài, ngài không thể làm gì được.

#4
Lee LOng

Lee LOng

    Trung sĩ

  • Thành viên
  • 174 Bài viết

Ta có:

$T=\sum \frac{1}{\frac{b^{4}+c^{4}}{a}+1}$
$\frac{b^{4}+c^{4}}{a}+1\geq \frac{(b^{2}+c^{2})^{2}}{2a}+1\geq \frac{(b^{2}+c^{2}).bc}{a}+1=\frac{b^{2}+c^{2}}{a^{2}}+1=\frac{a^{2}+b^{2}+c^{2}}{a^{2}}$
$\Rightarrow T\leq 1$


#5
longatk08

longatk08

    Sĩ quan

  • Thành viên
  • 350 Bài viết

Câu 5:

 

Ta chứng minh Max là $1$

 BĐT cần chứng minh tương đương với:

$\sum \frac{(b^4+c^4)}{b^4+c^4+a}\geq 2$

Áp dụng Cauchy-Schwarz thì.

$VT\geq \frac{(\sum \sqrt{b^4+c^4})^2}{\sum (b^4+c^4+a)}$

Ta cần chứng minh:

$\sum 2\sqrt{(b^4+a^4)(a^4+c^4)}\geq 2(a^4+b^4+c^4)+a+b+c$

 

LẠi áp dụng C-S trong căn thì BĐT này tương đương với:

$(ab)^2+(bc)^2+(ac)^2\geq a+b+c$

Đồng bậc thì được BĐT đúng


Bài viết đã được chỉnh sửa nội dung bởi longatk08: 02-06-2015 - 22:15


#6
LoveMath213

LoveMath213

    Binh nhì

  • Thành viên
  • 16 Bài viết

Ta có $ b^4+c^4\geqslant \dfrac{(b^2+c^2)^2}{2}\geqslant bc(b^2+c^2) $
Nên $ b^4+c^4+a\geqslant bc(a^2+b^2+c^2) \Rightarrow \dfrac{a}{b^4+c^4+a}\leqslant \dfrac{a}{bc(a^2+b^2+c^2)}=\dfrac{a^2}{a^2+b^2+c^2}$.
Chứng minh tương tự ta có $ \dfrac{b}{c^4+a^4+b}\leqslant \dfrac{b^2}{a^2+b^2+c^2} $ và $ \dfrac{c}{a^4+b^4+c}\leqslant \dfrac{c^2}{a^2+b^2+c^2} $.
Do đó $ P\leqslant 1 $ hay GTLN của $ P $ bằng 1 đạt tại $ a=b=c=1 $.



#7
LoveMath213

LoveMath213

    Binh nhì

  • Thành viên
  • 16 Bài viết

Câu 5.

1) Điều kiện $ x^2-2x-2\geqslant 0, x\leqslant 2 \Rightarrow x\leqslant 1-\sqrt{3}$.
Biến đổi phương trình thành \[ \sqrt{3x^2-6x-6}=3(2-x)^2\sqrt{2-x}+(7x-19)\sqrt{2-x} \Leftrightarrow \sqrt{3x^2-6x-6}=(3x^2-5x-7)\sqrt{2-x}\]
Đặt $ \sqrt{3x^2-6x-6}=u $ và $ \sqrt{2-x}=v $ ta có $ u,v\geqslant 0 $ và $ 3x^2-5x-7=u^2-v^2+1 $ và ta được phương trình \[ u=(u^2-v^2+1)v\Leftrightarrow (u-v)(v^2+uv-1)=0 .\]
Do $x\leqslant 1-\sqrt{3}\Rightarrow v^2=2-x\geqslant 1+\sqrt{3}>1 $ nên $ v^2+uv-1>0 $ và ta có $ u=v\Leftrightarrow 3x^2-5x-8=0 $, đối chiếu với điều kiện ta được $ x=-1 $ thỏa mãn.



#8
hoctrocuaHolmes

hoctrocuaHolmes

    Thượng úy

  • Thành viên
  • 1013 Bài viết

Đây là đề thi Toán chung của trường THPT Lê Hồng Phong Nam Định các bạn xem thử

Làm mấy bài phù hợp trình độ của em thôi  :icon6:

1.1)Biểu thức $\sqrt{x+1}+\sqrt{x-3}$ được xác định $\Leftrightarrow \left\{\begin{matrix} x+1\geq 0 & \\ x-3\geq 0 & \end{matrix}\right.$

$\Leftrightarrow \left\{\begin{matrix} x\geq -1 & \\ x\geq 3 & \end{matrix}\right. \Leftrightarrow x\geq 3$

2)Kết quả bằng $2$

2.1)$Q=(\frac{1}{\sqrt{x}-1}-\frac{2}{x-1}).(\frac{x+\sqrt{x}}{\sqrt{x}+1}-\frac{1-\sqrt{x}}{\sqrt{x}-x})$

$=\frac{1}{\sqrt{x}+1}.(\sqrt{x}-\frac{1}{\sqrt{x}})=\frac{x-1}{x+\sqrt{x}}$

2)$Q=-1\Leftrightarrow \frac{x-1}{x+\sqrt{x}}=-1\Leftrightarrow 2x+\sqrt{x}-1=0\Leftrightarrow (2\sqrt{x}-1)(\sqrt{x}+1)=0\Leftrightarrow 2\sqrt{x}-1=0\Leftrightarrow x=\frac{1}{\sqrt{2}}$     (vì $x>0$ )

3.1)Thay $m=3$ vào $PT (1)$ ta có 

$x^{2}-2(3-1)x+3^{2}-6=0\Leftrightarrow x^{2}-4x+3=0\Leftrightarrow (x-1)(x-3)=0\Leftrightarrow \begin{bmatrix} x=1 & \\ x=3& \end{bmatrix}$

2)Để $PT (1)$ có nghiệm thì

$\Delta =[2(m-1)]^{2}-4(m^{2}-6)\geq 0\Leftrightarrow 28-8m\geq 0\Leftrightarrow m\leq \frac{7}{2}$ 

Theo hệ thức Viet ta có $\left\{\begin{matrix} x_{1}+x_{2}=2(m-1) & \\ x_{1}.x_{2}=m^{2}-6 & \end{matrix}\right.$

Ta có $x_{1}^{2}+x_{2}^{2}=(x_{1}+x_{2})^{2}-2x_{1}.x_{2}=[2(m-1)]^{2}-2(m^{2}-6)=2m^{2}-8m+16=16\Leftrightarrow 2m^{2}-8m=0\Leftrightarrow \begin{bmatrix} 2m=0 & \\ m-4=0 & \end{bmatrix}$

$\Leftrightarrow \begin{bmatrix} m=0 (KTM) & \\ m=4 (TM)& \end{bmatrix}$

Vậy,$m=4$ là giá trị thoả mãn



#9
VASILE CIRTOAJE

VASILE CIRTOAJE

    Binh nhất

  • Banned
  • 21 Bài viết

câu hệ mình chỉ làm được đến đây thôi nha

 từ pt1   $(x-y+3)^{2}=\frac{y}{x+2} \Rightarrow (x-y+3)^{2}-1=\frac{y}{x+2}-1 \Leftrightarrow (x-y+2)(x-y+4)=-\frac{x-y+2}{x+2}$

    rồi giải dần có lẽ ra



#10
LoveMath213

LoveMath213

    Binh nhì

  • Thành viên
  • 16 Bài viết

Câu hệ phương trình.

Điều kiện $ x\geqslant -2, y\geqslant 0 $.
Đặt $ \sqrt{x+2}=u, \sqrt{y}=v $ ta có $ u,v\geqslant 0 $.
Phương trình thứ nhất thành $ u(u^2-v^2+1)=v\Leftrightarrow (u-v)(u^2+uv+1)=0\Leftrightarrow u=v $. (do $ u,v\geqslant 0 $ ).
Do đó $ y=x+2 $ thay vào pt thứ hai ta được $ 2x^2+5x-7=0 $ từ đó có $ x=1$, $x=-\dfrac{7}{2} $ (loại).
Hệ có nghiệm $ (1;3) $


Bài viết đã được chỉnh sửa nội dung bởi LoveMath213: 03-06-2015 - 08:41


#11
devilloveangel

devilloveangel

    Hạ sĩ

  • Thành viên
  • 78 Bài viết

             Sở GD-ĐT Nam Định                                                                        ĐỀ THI TUYỂN SINH VÀO LỚP 10 CHUYÊN

Trường THPT Chuyên Lê Hồng Phong                                                                           NĂM HỌC : 2015 - 2016

                                                                                                                                       MÔN THI : TOÁN CHUYÊN

 

 

Câu 1: ( 2.0 điểm ) 

1)  Cho đa thức $P(x) = ax^{2} + bx + c$ . biết $P(x)$ chia cho $x + 1$ dư $3$ , chia cho $x$ dư $1$ và chia cho $x - 1$ dư $5$ , hãy tìm các hệ số $a,b,c$ 

2) Cho các số $a,b,x,y$ thỏa mãn $ab\neq 0$ , $a+b\neq 0$ , $\frac{x^{4}}{a} + \frac{y^{4}}{b} = \frac{1}{a+b}$ , $x^2+y^2=1$ , Chứng minh rằng : 

a) $ay^2=bx^2$

b) $\frac{x^{200}}{a^{100}} + \frac{y^{200}}{b^{100}} = \frac{2}{(a+b)^{100}}$ 

Câu 2: ( 2.5 điểm ) 

1) Giải hệ phương trình : $\left\{\begin{matrix} ( y -x)(y-x-4) = x^{2} - 4x \\ x(y-4) + 4\sqrt[3]{x-y} = 6 \end{matrix}\right.$

2) Giải phương trình : $3(x+1)\sqrt{x^{2} + x +3} - 3x^2 - 4x -7 = 0$

Câu 3: ( 3.0 điểm ) 

Cho hai đường tròn (O1) và (O2) tiếp xúc ngoài với nhau tại M , Một đường thẳng cắt đường tròn (O1) tại 2 điểm phân biệt A và B và tiếp xúc đường tròn (O2) tại E ( B nằm giữa A và E ) , Đường thẳng EM cắt đường tròn (O1) tại điểm J khác M , gọi C là điểm thuộc cung MJ không chứa A,B của đường tròn (O1) ( C khác M và J ) , Kẻ tiếp tuyến CF với đường tròn (O2) ( F là tiếp điểm ) sao cho các đoạn thẳng CF , MJ không cắt nhau , Gọi I là giao điểm các đường thẳng JC và EF , K là giao điểm khác A của AI và đường tròn (O1) Chứng minh rằng : 

1) Tứ giác $MCFI$ là tứ giác nội tiếp và $JA = JI  = \sqrt{JE.JM}$

2) $CI$ là phân giác góc ngoài tại  $C$ của tam giác $ABC$.

3) $K$ là tâm đường tròn ngoại tiếp tam giác $BCI$.

Câu 4: ( 1.0 điểm ) 

Tìm các số tự nhiên $x,y$ thỏa mãn : 

$(2^x + 1)(2^x + 2)(2^x + 3)(2^x + 4) - 5^y = 11879$ 

Câu 5: ( 1.5 điểm ) 

1) Trong mặt phẳng cho tập $S$ gồm $8065$ điểm đôi một phân biệt mà diện tích của mỗi tam giác có $3$ đỉnh thuộc tập $S$ đều không lớn hơn $1$ ( quy ước nếu $3$ điểm thẳng hàng thì diện tích của tam giác tạo bởi $3$ điểm này bằng $0$) . CMR tồn tại $1$ tam giác $T$ có diện tích không lớn hơn $1$ chứa ít nhất $2017$ điểm thuộc tập $S$ ( mỗi điểm trong số $2017$ điểm đó nằm trong hoặc nằm trên cạnh của tam giác $T$).

2) Cho ba số dương $a,b,c$. Chứng minh bất đẳng thức:

$\frac{4a^2 + (b-c)^2}{2a^2 + b^2 + c^2} + \frac{4b^2 + (c-a)^2}{2b^2 + c^2 + a^2} + \frac{4c^2 + (a-b)^2}{2c^2 + a^2 + b^2} \geqslant 3.$

 

------ HẾT------
Xóa bớt latex cho bài dễ nhìn


Bài viết đã được chỉnh sửa nội dung bởi devilloveangel: 03-06-2015 - 18:59

Imagination rules the world.


#12
devilloveangel

devilloveangel

    Hạ sĩ

  • Thành viên
  • 78 Bài viết

Câu 1a) 

vì P(x) chia cho x + 1 dư 3 , chia cho x dư 1 và chia cho x - 1 dư 5 hay

P(x) - 3 chia hết cho x + 1 

P(x) - 1 chia hết cho x 

P(x) - 5 chia hết cho x -1 

Ta có hệ phương trình : 

$ax^2 + bx + c$ - 3 = 0 với x = -1 

$ax^2 + bx + c$ - 1 = 0 với x = 0

$ax^2 + bx + c$ - 5 = 0 với x = 1 

hay : 

a - b + c - 3 = 0 

c = 1 

a + b + c - 5 = 0  

hay 

a - b = 2

a + b = 4 => a = 3 , b = 1 , c = 1 thõa mãn điều kiện => P(x) = $3x^2 + x + 1$ = 0


Imagination rules the world.


#13
Lee LOng

Lee LOng

    Trung sĩ

  • Thành viên
  • 174 Bài viết

4,$(4^{x}+5.2^{x}+5)=5^{y}+11880$

$=>y=0=>x=3$


#14
devilloveangel

devilloveangel

    Hạ sĩ

  • Thành viên
  • 78 Bài viết

Câu 2 :

1) : 

Vế đầu pt viết lại :

(y-x)(y-4) - x(y-x) = x(x-4) <=> (y-x)(y-4) = x(x-4+y-x) = x(y-4) <=> (y-4)(y - 2x) = 0 => y = 2x hoặc y = 4 

Với y =4 => x =7,375

Với y = 2x thế vào phương trình dưới giải ra x => y 

 

2) 

đặt x+1 = a , $\sqrt{x^2 + x + 3}$ = b có phương trình : $3ab - a^2 - 2b^2$ = 0 hay $(a-2b)(a-b)=0$ => a = 2b hoặc a = b => x


Bài viết đã được chỉnh sửa nội dung bởi devilloveangel: 03-06-2015 - 14:06

Imagination rules the world.


#15
dogsteven

dogsteven

    Đại úy

  • Thành viên
  • 1567 Bài viết

Bài 5.

(a) Xét bao lồi cho tập hợp $8065$ điểm trên. Tồn tại hai đỉnh của bao lồi là $A,B$ sao cho qua $A,B$ dựng được hai đường thẳng tựa vuông góc với $AB$

Xét điểm $D,C$ thuộc bao lồi nói trên khác phía so với $AB$ sao cho qua $C,D$ dựng được đường thẳng tựa song song với $AB$

Các đường thẳng tựa này cắt nhau tạo thành một hình chữ nhật.

Lấy đường chéo chia hình chữ nhật đó thành $4$ tam giác nhỏ.

Diện tích của hình chữ nhật đó là $AB.[d(C,AB)+d(D,AB)]=2S(ABC)+2S(ABD)\leqslant 4$

Do đó diện tích mỗi tam giác sẽ nhỏ hơn hoặc bằng $1$

Xét tam giác chứa nhiều điểm nhất thì tam giác đó thỏa mãn.

P.s. Kiểm tra lại giúp em.


Bài viết đã được chỉnh sửa nội dung bởi dogsteven: 03-06-2015 - 13:19

Quyết tâm off dài dài cày hình, số, tổ, rời rạc.


#16
Pham Quoc Thang

Pham Quoc Thang

    Trung sĩ

  • Thành viên
  • 160 Bài viết

Bất đẳng thức cần chứng minh tương đương với:$\sum\frac{(b+c)^2}{2a^2+b^2+c^2} \leq 3 $
Áp dụng bất đẳng thức Cauchy Schwarz ta có:$\frac{b^2}{b^2+a^2}+\frac{c^2}{c^2+a^2} \geq \frac{(b+c)^2}{2a^2+b^2+c^2} $
Tương tự,cộng lại ta thu đc đpcm



#17
Pham Quoc Thang

Pham Quoc Thang

    Trung sĩ

  • Thành viên
  • 160 Bài viết

Câu 2:
2)Bình phương 2 vế ta được:$9(x^2+2x+1)(x^2+x+3)=(3x^2+4x+7)^2$
Phương trình này có thể thu gọn đưa về phương trình bậc 3.Tìm được nghiệm duy nhất là x=2
Vậy S={2}


Bài viết đã được chỉnh sửa nội dung bởi Pham Quoc Thang: 03-06-2015 - 14:04


#18
arsfanfc

arsfanfc

    Sĩ quan

  • Thành viên
  • 377 Bài viết

             Sở GD-ĐT Nam Định                                                                        ĐỀ THI TUYỂN SINH VÀO LỚP 10 CHUYÊN

Trường THPT Chuyên Lê Hồng Phong                                                                           NĂM HỌC : 2015 - 2016

                                                                                                                                       MÔN THI : TOÁN CHUYÊN

 

 

 

Câu 2: ( 2.5đ ) 

 

2) Giải phương trình : $3(x+1)\sqrt{x^{2} + x +3} - 3x^2 - 4x -7 = 0$

 

 

ĐK:$ x^2+x+3 \geq 0 => x.....$

$PT<=> 3(x+1)\sqrt{x^2+x+3}-3(x^2+x+3)-(x-2)=0$

$<=> 3(\sqrt{x^2+x+3})(x+1-\sqrt{x^2+x+3})-(x-2)=0$

$<=> 3\sqrt{x^2+x+3}(\frac{x-2}{(x+1)+\sqrt{x^2+x+3}})-(x-2)=0$

$<=> (x-2)(\frac{3\sqrt{x^2+x+3}}{x+1+\sqrt{x^2+x+3}}-1)=0$

$=> x=2$ hoặc $\frac{3\sqrt{x^2+x+3}}{x+1+\sqrt{x^2+x+3}}=1 <=>3x^2+2x+11=0$ (vô nghiệm )


Bài viết đã được chỉnh sửa nội dung bởi arsfanfc: 03-06-2015 - 14:13

~YÊU ~


#19
Pham Quoc Thang

Pham Quoc Thang

    Trung sĩ

  • Thành viên
  • 160 Bài viết

Câu 1
2)a)
$\frac{x^4}{a}+\frac{y^4}{b}=\frac{1}{a+b} \Leftrightarrow \frac{b.x^4+a.y^4}{ab}=\frac{1}{a+b}$
$\Leftrightarrow ab=(b.x^4+a.y^4)(a+b)$
Mà $ab(x^4+y^4)+(ay^2)^2+(bx^2)^2=ab(1-2x^2y^2)+(ay^2)^2+(bx^2)^2$
Nên $(ay^2-bx^2)^2=0$
Suy ra đpcm



#20
devilloveangel

devilloveangel

    Hạ sĩ

  • Thành viên
  • 78 Bài viết

Câu 1) 2b) , đặt $x^{200} = X^2$  , đặt $y^{200} = Y^2$ , $a^{100} = A , b^{100} = B$ , biến đổi và áp dụng kết quả thu được từ câu 1) 2a) ta có đpcm


Bài viết đã được chỉnh sửa nội dung bởi devilloveangel: 03-06-2015 - 15:04

Imagination rules the world.





1 người đang xem chủ đề

0 thành viên, 1 khách, 0 thành viên ẩn danh